Population inversion, 2-level system and 3-level system

  Рет қаралды 42,669

Laser: Fundamentals and Applications

Laser: Fundamentals and Applications

Күн бұрын

Пікірлер: 47
@melbinsiby250
@melbinsiby250 2 жыл бұрын
those who are stuck at where 2Bp came from, dont forget that in the equation it is Ntotal =N1+N2. So in rate eqn if you put N1 as Ntotal-N2 and then find eqn for N2 you will get the 2 in den.
@rameshwar1272
@rameshwar1272 Жыл бұрын
At 17:00 sir has taken the equilibrium condition when dN2/dt will be zero and we get the N2 at equilibrium by this formula...of course you have to use the fact N1=Ntotal-N2 in previous equation to get this form.... And you will see next that this N2 can never be greater than the N1 or half of N total that's why we can't get laser action in 2 level system
@aaqibali9349
@aaqibali9349 6 жыл бұрын
sir if N2 =0 how spontaneous emission take place at 17:48 or how N2 = to something if it is already zero
@mm__1659
@mm__1659 3 жыл бұрын
This is actually an initial condition......but what he did is that at steady state (dN2/dt=0)...and extract the value of N2 and algebraic manipulation
@corporalcharan9892
@corporalcharan9892 2 жыл бұрын
@@mm__1659 why is it in steady state?
@rameshwar1272
@rameshwar1272 Жыл бұрын
@@mm__1659 did you mean the thermal equilibrium state when the rate of change will be fixed up... And thanks I'm also thinking about the extreme state or the fixed steady state.
@mahibabu6789
@mahibabu6789 7 жыл бұрын
Nice lecture, Can you please explain how the N2/N1 equation derived by assuming the condition at time t=o, N2=0 and N1=Ntotal thank you
@aaqibali9349
@aaqibali9349 6 жыл бұрын
same question kindly tell me if you get answered
@giancarloplazzi2364
@giancarloplazzi2364 5 жыл бұрын
@@aaqibali9349 when gain equals to loss , there is no emission .therefore , dN2/ dt =0 ( not N2=0)
@heenasingh9355
@heenasingh9355 5 жыл бұрын
@@giancarloplazzi2364 but how we write 2B in denominator it must be A+Brho
@raviraturi6034
@raviraturi6034 4 жыл бұрын
Study 1st order linear differential equation.
@jake1967
@jake1967 2 жыл бұрын
​@@heenasingh9355 Before going into conditions, change N1 = Ntotal - N2. Then put dN2/dt = 0. Finally, find an expression for N2.
@alkanedust3848
@alkanedust3848 3 жыл бұрын
What is the significance of drawing level 3 down, is it a high energy state only then why there is no 1->3 ?
@gobindachakraborty245
@gobindachakraborty245 6 жыл бұрын
You guys are doing a great job!!!I
@shideshpatil4796
@shideshpatil4796 4 жыл бұрын
Sir which book you are following?
@dr.s.esakkimuthu6741
@dr.s.esakkimuthu6741 4 жыл бұрын
when you explain please clearly explain the intermediate steps and their physical meaning. since in A. Ghatak they derived in different way and in B.B.Laud too.
@rohandevaki4349
@rohandevaki4349 5 жыл бұрын
how can we have a spontaneous emission and stimulated emission from 3 to 1 in a three level system
@putikibabuji1453
@putikibabuji1453 3 жыл бұрын
Sir a small doubt N2=0 then how can write again N2/N1= something
@rohandevaki4349
@rohandevaki4349 5 жыл бұрын
please provide the link your previous lecture here
@rohandevaki4349
@rohandevaki4349 5 жыл бұрын
how did u get the eqn n2=ntotalxPxB/A+2xBxP at 17:26
@crazygeek5825
@crazygeek5825 4 жыл бұрын
Yeah dude
@ankitanain3555
@ankitanain3555 4 жыл бұрын
N1/N2=B*rho +A/B*rho Add 1 both sides Take lcm and then again do reciprocal
@alokkumar3703
@alokkumar3703 4 жыл бұрын
@@ankitanain3555 nice job man ,bright student
@mohamedjamal6023
@mohamedjamal6023 4 жыл бұрын
@@ankitanain3555 How?
@ghanashyamutube
@ghanashyamutube 4 жыл бұрын
@@mohamedjamal6023 Do d same as Ankita said and simply take N1+N2 as Ntotal, then u will get.
@rohandevaki4349
@rohandevaki4349 5 жыл бұрын
what is meant by dark state in a three level system at 27:35
@alenvoka2391
@alenvoka2391 3 жыл бұрын
thank you master.
@ARCSTREAMS
@ARCSTREAMS 3 жыл бұрын
i dont understand how you proved that N2 can not be bigger than N1? all you did was say it can't?
@BlackBuzzTV
@BlackBuzzTV 3 жыл бұрын
I don't actually know if this is proper demonstration but if you consider the N2/N1 ratio to be described by the Boltzmann equation (used previously in the course) you have N2/N1=e^-X which cannot be greater than 1. This is also pretty much understable as a gain medium cannot be pumped to population inversion as when it reaches the N2=N1 ratio, it becomes transparent to that wavelength, in other words, the rate of absorption would be the same as the rate of stimulated emission.
@ARCSTREAMS
@ARCSTREAMS 3 жыл бұрын
@@BlackBuzzTV transparent to what pump wavelength? so is it still possible to get a laser beam even if n2 =n1? , i forgot what laser this was all referring to, but i get head wrapped with all this confusing stuff no matter how someone tries to explain things ,thx anyways
@BlackBuzzTV
@BlackBuzzTV 3 жыл бұрын
@@ARCSTREAMS Transparent to the N2-N1 energy related frequency (in this case both the pumping frequency and lasering frenquency as they are the same (which is the problem by the way)). In the case of a two-level LASER, one cannot reach population inversion as when the N1=N2 rate is reached, the stimulated emission phenomenon rate is equivalent to the absorption rate. Imagine this on two atoms only : if one is excited and the other one isn't, you will send a photon on it, a stimulated emission phenomenon will occur (on the excited atom), so you have two photons now, and when you reach the second atom (unexcited one) one of the two photons you have will be absorbed. The result of this is that you will have the same amount of photons (1) and the same amount of excited atoms (1).
@ARCSTREAMS
@ARCSTREAMS 3 жыл бұрын
@@BlackBuzzTV ok i understood the two atoms example for a 2 level but still not getting the transparent thing ,you said botht he pump and and simulating freq(ie wavelenght) are the same and this is a problem,can you please give an example of why or did you do this already lol? also i always thought that the pump freq is always different that the lasing freq and that the pump will have more energy than the lasing being created so for example your pump is 808nm and lasing is 1064nm ,well the 1064 can only stimulate exited electrons and can not pump them (ie absoprtion then spontaneous emission of 1064) ,are you saying i can use the same freq to pump as that of the lasing being created? so i can use for example 1064 pump to create 1064 lasing? or are there other examples where this happens?
@karansharma6835
@karansharma6835 5 жыл бұрын
is this in syllabusof 1st year. Please answer this
@gowthamm3987
@gowthamm3987 4 жыл бұрын
Yes
@drscott1
@drscott1 2 жыл бұрын
👍🏼
3-level System and 4-level system
34:36
Laser: Fundamentals and Applications
Рет қаралды 41 М.
Components of LASERs
27:57
Laser: Fundamentals and Applications
Рет қаралды 22 М.
How do Cats Eat Watermelon? 🍉
00:21
One More
Рет қаралды 14 МЛН
Mode Locking
33:24
Laser: Fundamentals and Applications
Рет қаралды 32 М.
Modes of LASER cavity and standing waves
31:26
Laser: Fundamentals and Applications
Рет қаралды 38 М.
How Does a Laser Work? Quantum Nature of Light - [3]
22:44
Math and Science
Рет қаралды 98 М.
Q-switching
30:08
Laser: Fundamentals and Applications
Рет қаралды 36 М.
Einsteins Concept of stimulated emission
22:19
Laser: Fundamentals and Applications
Рет қаралды 30 М.
Calculation of Einsteins coefficient
31:36
Laser: Fundamentals and Applications
Рет қаралды 29 М.
Laser Physics - Stimulated Emission & Einstein Coefficients | Three Level Laser
41:11